there are 45 marbles in a blue jar. the blue jar has 9 times as many marbles as a red jar. let m be the number of marbles in the red jar.
Write an equation that relates the number of marbles in the two jars. Use 9, m, and 45​

Answers

Answer 1

Answer:

9m = 45

Step-by-step explanation:

if the blue jar has 9 times as many marbles as the red jar (which is 45 marbles), then that means that:

9m = 45

where m is the number of marbles in the red jar.


Related Questions

3. A jet traveled at an average speed of 681 kilometers an hour. At that rate, how far did the jet go in 7 1/4 hours?​

Answers

[tex]\stackrel{mixed}{7\frac{1}{4}}\implies \cfrac{7\cdot 4+1}{4}\implies \stackrel{improper}{\cfrac{29}{4}} \\\\[-0.35em] ~\dotfill\\\\ \begin{array}{ccll} K ms&hours\\ \cline{1-2} 681 & 1\\ x& \frac{29}{4} \end{array} \implies \cfrac{681}{x}=\cfrac{1}{~~ \frac{29}{4}~~}\implies \cfrac{684}{x}=\cfrac{4}{29} \\\\\\ 19836=4x\implies \cfrac{19836}{4}=x\implies 4959=x[/tex]

Ali wants to surprise his wife Sara by presenting her some flowers, when he returns back from a work tour. He plans to spend exactly $24 on a bunch of exactly two dozen flowers. Sara loves lilies, roses and daisies. At the flower market they are selling lilies for $3 each, roses for $2 each, and daisies $0.50 each. How many flowers of each type can Ali buy?

Answers

Answer:

he can buy 8 of lilies and 12 of rose and 48 of daisies

Ali can buy 3 flowers of lilies, 3 flowers of roses, and 18 flowers of daisies.

Suppose Ali buys x flowers of lilies, y flowers of roses, and z flowers of daisies.

According to the question

x+y+z = 24......(1)

3x+2y+0.5z = 24.......(2)

Here, we have three variables but only two equations.

2(2)-(1) gives, 5x+3y = 24......(3)

Since, x, y and z can take only integer values.

What are integers?

Numbers that are not fractions called integers.

So, from (3) only x=3 and y=3 is possible

So, z= 24-x-y

z=24-3-3 = 18

So, Ali can buy 3 flowers of lilies, 3 flowers of roses, and 18 flowers of daisies.

Therefore, Ali can buy 3 flowers of lilies, 3 flowers of roses, and 18 flowers of daisies.

To get more about linear equations visit:

https://brainly.com/question/14323743

This figure represents a compartment in a container for storing sewing supplies. Another compartment is identical except that it is 3 centimeters taller.

What is the difference in volume of the two compartments?

Enter your answer in the box.
cm³
Right trapezoidal prism. The height of the trapezoid base is labeled 7 cm. The larger base is labeled 15 cm and the shorter base is labeled 5 cm. The two legs of the trapezoid are labeled 10 cm. The height of the prism is labeled 8 cm.

cm³


help please

Answers

Answer:

140

explanation:

I took the test

Answer:

210 cm

Step-by-step explanation:

If the trapezoid in the picture is the one mentioned in the problem, then the answer is 210 cm. I did the test.

Suppose if, instead of depositing the $600 all at once, you deposit nothing at the beginning and you divide up the $600 into 12 envelopes each with $50. Find the balance after one year if you deposit one $50 envelope each month, all year, into an account that pays 5% APR with monthly compounding.

Answers

The balance after one year if you deposit one $50 envelope each month, all year is  $613.95.

What is the balance after one year?

The balance after 1 year can be determined using this formula:

Amount deposited monthly x annuity factor

Annuity factor = {[(1+r)^n] - 1} / r

Where:

r = interest rate = 5% /12 = 0.417%n = number of years = 1 x 12 = 12

Annuity factor = 1.004167^12 - 1 / 0.00417 = 12.279082

Balance = 12.279082 x 50 = $613.95

To learn more about annuities, please check: https://brainly.com/question/24108530

Michael has a bag with 2 red marbles, 4 white marbles and 3 purple marbles. He will randomly select 2 marbles from the bag one at a time without
replacement. What is the probability that he will choose a red marble first and then a white marble?
O 13/18
8/81
O 1/9
O 2/3

Answers

Answer:

1/9

Step-by-step explanation:

Given:

2 red marbles

4 white marbles

3 purple marbles

He will randomly select 2 marbles from the bag one at a time without

replacement.

To Find:

What is the probability that he will choose a red marble first and then a white marble?

Solution:

Since red marbles =2

White = 4

Total = 2+4+3 = 9

Select first time = 2/9

Select second time = 4/8

And then = Multiply

2/9 * 4/8 = 8/72

8/72 Simplify

1/9

[RevyBreeze]

Shawn is typing paper for a class. He can type 1 6/11 pages in 1/3 of an hour. How many pages can shawn type in in one hour

Answers

Answer:

4 7/11

Step-by-step explanation:

1 6/11 * 3 = 3 18/11 = 4 7/11

What are the y-intercepts of this conic section?



(0, −4) and (0, 4)

(0, −2) and (0, 2)

no y-intercepts

(−2, 0) and (2, 0)

Answers

Step-by-step explanation:

the second one, of course.

a y-intercept is a point where x = 0.

the 2 options are clearly having y-values of -2 and +2.

How many different ways can 6 different positions be filled by 12 applicants?

Answers

Answer:665280

Step-by-step explanation:

12•11•10•9•8•7=665280

120 feet is to 150 feet as 8 feet is to ten feet write the proportion HELP PLEASE!!! AND PLEASE SHOW HOW TO DO IT

Answers

Answer:

Step-by-step explanation:

120/150  = 0.8

8/10 = 0.8

8x15=120

10x15=150

What length is X
3.0000
4.0000

Answers

kinda confused not sure if we have an equation or other numbers to help answer

19. Given that angle BFC = 58°. Angle BXG = 48° and angle CBF 22 Find () 2BGX (iv) BCG (i) 2BGF (iv) ZBFG (ii) BCF H EE 58 48° D C 220 A- Figure 5.74​

Answers

The unknown angles in the cyclic quadrilateral is as follows:

∠BGX = 74° (sum of angles in a triangle)

∠BGF = 180° (opposite angles of cyclic quadrilateral are supplementary)

∠BCF = 100°(sum of angles in a triangle)

∠BCG  = 26°

∠BFG = 22°

Cyclic Quadrilateral

A cyclic quadrilateral has all its angles equal to 360 degrees. The sum of angles in a cyclic quadrilateral is equals to 360 degrees.

Let's find the missing angles as follows:

∠BGX = 180 - 48 - 58 = 74° (sum of angles in a triangle)

∠BGF = 180 - 100 = 80° (opposite angles of cyclic quadrilateral are supplementary)

∠BCF = 180 - 22 - 58 = 100°(sum of angles in a triangle)

∠BCG = 100 - 74 = 26°

∠BFG ≅ CBF = 22°(alternate angles)

learn more on angles here: https://brainly.com/question/19430381

how do you find the square root for 9cm ​

Answers

Square Root of 9:-

[tex]\rightarrow[/tex]Square root of 9 is a whole number that is multiplied by itself to get the original number. The actual value of square root of 9 is 3.

√9 = ±3

[tex]\rightarrow[/tex]If we square +3 or -3, the resulting value will be equal to 9.

[tex]\sf{(+3)^2 = 9}[/tex]

[tex]\sf{(-3)^2 = 9}[/tex]

How to find?:-

[tex]\rightarrow[/tex]Number 9 is an odd number and not a prime number. Prime numbers have only two factors, 1 and the number itself. But as we know, 9 have three multiple factors, 1, 3 and 9 itself. Thus, we can write the number 9 in the form of multiple as;

[tex]\sf{1 × 9 = 9}[/tex][tex]\sf{3 × 3 = 9}[/tex][tex]\sf{9 × 1 = 9}[/tex]

[tex]\rightarrow[/tex]You know that number 9 is a multiple of 3 or when 3 is multiplied by itself, we get the number 9. Therefore, we can write the square root of 9 as;

➾[tex]\sf{\sqrt{9}}[/tex]

➾[tex]\sf{\sqrt{3×3}}[/tex]

➾[tex]\sf{\sqrt{3^2}}[/tex]

[tex]\rightarrow[/tex]The square cancels the square root of a number. Therefore, if we cancel the square root with a square in the above expression, we get,

➾[tex]\sf{\sqrt{9}}[/tex] = ±3

__________________________________

Hope it helps you:)

Solve for x.

x−9=−524

Enter your answer as a mixed number in simplest form by filling in the boxes.

x = $$

Answers

X is equal to 515 that’s the correct answer after collecting like terms

Answer:

The answer is 1 7/8

Step-by-step explanation:

I did the test. Screenshot here;

Hope this helps!

Help Picture below problem 17

Answers

Answer:

139°

Step-by-step explanation:

18°+23°+?= 180°{sum of angles on a triangle}

41°+?° = 180°

?° =180-41=139°

How many inches are in 6 3/4 feet? Enter only a number. Do not include units.

Answers

Answer:

81 inches

Step-by-step explanation:

I believe this is it…

The answer is 81

How to Convert Feet to Inches
To convert a foot measurement to an inch measurement, multiply the length by the conversion ratio.

Since one foot is equal to 12 inches, you can use this simple formula to convert:

inches = feet × 12
The length in inches is equal to the feet multiplied by 12.

The sum of two integers is 72 and their difference is 8. Find the integers

Answers

The answer is 9 it’s correct
Hence the 2 integers are 32 and 40

The polynomial below is a perfect square trinomial of the form A^2 - 2AB + B^2

9x^2 - 12x + 4

A. True

B. False

Answers

Answer:

True

Step-by-step explanation:

[tex]9x^2-12x +4 \\\\=(3x)^2 - 2\cdot 3x \cdot 2+2^2\\\\=(3x-2)^2[/tex]

Which of the following would describe Slope?
A Rise/Run


B The rate of change


C The steepness of a line


D All of the above

Answers

I guess A because rise/run is one graph and that’s how you plug the slope on the graph

Answer:

Rise/Run (A)

Step-by-step explanation:

The slope of a line measures the steepness of the line. Most of you are probably familiar with associating slope with "rise over run". Rise means how many units you move up or down from point to point. On the graph that would be a change in the y values. Run means how far left or right you move from point to point.

Given that the width is 4.5 units and the length can be represented by x + 3, which answer choice shows the correct expression of the area below?
Please just explain step by step how you got the answer.
!!! I WILL GIVE YOU 50 POINTS IF YOU SOLVE IT!!!

Answers

Step-by-step explanation:

the area of a rectangle is

length × width

that is in our case

(x + 3) × 4.5 = 4.5x + 4.5 × 3 = 4.5x + 13.5

remember, when multiplying 2 expressions, you multiply every term of one expression by every term of the other expression and then you add all these individual results up by considering the signs, of course.

Need assistance on this problem

Answers

Answer:

The height of the tower should be 120 m

Step-by-step explanation:

the process of splitting a numbers into equal parts or groups

Answers

Answer:

Division

Step-by-step explanation:

Dictionary Definition:

the action of separating something into parts or the process of being separated.

Hope that helps

A bag contains 9 green marbles, 6 purple marbles, 4 red marbles, 10 blue marbles, and 7 orange marbles. One marble is randomly selected from the bag.

What is the probability that the marble selected is blue or purple?

Answers

The probability that the marble selected is blue or purple is the likelihhood of selecting a blue or purple marble

The probability that the marble selected is blue or purple is 0.444

How to determine the probability?

From the question, we have:

Green = 9

Purple = 6

Red = 4

Blue = 10

Orange = 7

So, the total number of marbles is:

Total = 9 + 6 + 4 + 10 + 7

Total = 36

The number of blue or purple marbles is;

Blue or purple = 10 + 6

Blue or purple = 16

So, the probability that the marble selected is blue or purple

p = 16/36

Evaluate

p = 0.444

Hence, the probability that the marble selected is blue or purple is 0.444

Read more about probability at:

https://brainly.com/question/251701

what is the sum? complete the equation 3 1/7 + (-5 4/7)

Answers

Answer:

The answer is

[tex] - 2 \frac{3}{7} [/tex]

What is the value of n in the equation below?

Answers

Answer:

9

Step-by-step explanation:

12ⁿ⁻⁵ = 12⁴

n - 5 = 4

n = 9

What is the slope of the line y = − 4 , equals, minus, 4? Choose 1 answer:

Answers

Answer: the slope is 0

Step-by-step explanation:

Help help math math math

Answers

Answer: 5a+23

Step-by-step explanation: Let us combine like terms. -7a + 12a = 5a, and 17 + 6 = 23, thus it is 5a + 23.

Answer:

slay

Step-by-step explanation:

slay

Given the figure below , find the blues of x and z

Answers

Check the picture below.

h=-16t^2+320, how long will it take to hit the ground

Answers

Answer:

It will take 5 seconds

The data set below represents the different costs of a tablet at an electronic store. $298. $196, $248, $379, $319, $276, $198, $349, $319. What is the median?​

Answers

Answer:median of (298, 196, 248) - 196

median of (379, 319) - 349

median of (276,198, 349) - 198

Step-by-step explanation:

Answer:

298

Step-by-step explanation:

Given:

$298,$196, $248, $379, $319, $276, $198, $349, $319

Solve:

Put in order:

196, 198, 248, 276, 298, 319, 319, 349, 379

Median -

Arrange data values from lowest to highest value

The median is the data value in the middle of the set

If there are 2 data values in the middle the median is the mean of those 2 values.

Solution:

196, 198, 248, 276, 298, 319, 319, 349, 379

196, 198, 248, 276, 298, 319, 319, 349, 379

196, 198, 248, 276, 298, 319, 319, 349, 379

196, 198, 248, 276, 298, 319, 319, 349, 379

 Hence, the median is 298.

~lenvy~

Identify the correct weight to the nearest 1/8 pound, then reduce the fraction if possible.

A. 2 12/16 = 2 3/4 Ib
B. 2 2/8 = 2 1/4 Ib
C. 2 14/16 = 2 7/8 Ib
D. 2 1/2 Ib

Answers

The measured weight is (2 + 3/4) lb, so the correct option is A.

How to read the measure?

First, we can see that the needle is between the 2lb and 3lb mark, so we know that we have a measure between these two values.

Now, let's count the number of small lines between the needle and the 2lb mark. There are 12 marks (the 12th is below the needle). Then the measure will be:

(2 + 12/16) lb.

Now we want to simplify the fraction. If we divide both numerator and denominator by 4, we get:

12/16 = 3/4

Then the weight is:

(2 + 3/4) lb

Then the correct option is A.

If you want to learn more about weight, you can read:

https://brainly.com/question/11408596

Answer: Now, let's count the number of small lines between the needle and the 2lb mark. There are 12 marks (the 12th is below the needle). Then the measure will be:

(2 + 12/16) lb.

Now we want to simplify the fraction. If we divide both numerator and denominator by 4, we get:

12/16 = 3/4

Then the weight is:

(2 + 3/4) lb

Then the correct option is A.

Step-by-step explanation:

Other Questions
At the height of the northern empire, how far north did the empire stretch? How many solutions does each polynomial have? Dos puertas Vocabulario en contextoBehind one door is the hero waiting to marry the princess. Behind another is theenemy waiting to kidnap the princess. Answer the questions below and unscramblethe clue letters to help the princess choose the right door.PTE DOUA180I1. Quin tiene celos de la princesa y el hroc?2. La princesa y el hroe se casan porque estn3. Dnde vive el emperador y la princesa?on of Houghton Min CompanyBonus: Which door should the princess choose?234 ????????????????????? Identify major events leadingto the elimination of apartheid. State the following numbers using '+' or '_'. (a) 80 less than zero =(b) 76 more than zero=2.List all integers (a) from -8 to 4=(b) from -12 to -2= what is the area of a circle with a circumference of 250also I lost some of my friends on here and I cant find them and I haven't seen them since June :( Muscular Strength Training requires Which of these BESTdescribes the DNAantiparallel doublehelix?A. BOTH strands run ONLY in the3' to 5' direction.B. BOTH strands run in randomdirections.C. ONE strand runs in the 5' to 3'direction, and the other strandruns in the 3' to 5' direction. Cai tried to prove that \triangle FGH\cong \triangle HIJFGHHIJtriangle, F, G, H, \cong, triangle, H, I, J. Statement Reason 1 FG=HI=6FG=HI=6F, G, equals, H, I, equals, 6 Given 2 FH=HJ=4FH=HJ=4F, H, equals, H, J, equals, 4 Given 3 \overline{FG} \parallel \overline{HI} FG HI start overline, F, G, end overline, \parallel, start overline, H, I, end overline Given 4 \angle HFG\cong\angle JHIHFGJHIangle, H, F, G, \cong, angle, J, H, I When a transversal crosses parallel lines, alternate interior angles are congruent. 5 \triangle FGH\cong \triangle HIJFGHHIJtriangle, F, G, H, \cong, triangle, H, I, J Side-angle-side congruence What is the first error Cai made in his proof? Choose 1 answer: Choose 1 answer: (Choice A) A Cai used an invalid reason to justify the congruence of a pair of sides or angles. (Choice B) B Cai only established some of the necessary conditions for a congruence criterion. (Choice C) C Cai established all necessary conditions, but then used an inappropriate congruence criterion. (Choice D) D Cai used a criterion that does not guarantee congruence there are 374 students at a school. Each table in the cafeteria hold 16 students. What is the MINIMUM number of tables needed for all of the students to eat lunch at the same time How many units are there between point A (-3,80) and point B (-3,12) 6-7 skills practice solving radical equations and inequalities Why is it important to plan early for your retirement?That way you can choose the right major in college.In order to take advantage of the power of compounding.Planning early will allow you to invest in new companies.Taxes on retirement investments increase as you get older. Sequencing of Events: The table represents the number of miles to the nearest airport. Find the median.# OF MILES: 20, 22, 24, 26 FREQUENCY: 3, 2, 1, 4 Amir is a collector. He currently has 397 rare stamps. Amir wants to double his collection. How many stamps will be in Amir's collection? stamps Do anjas grandparents go to theresienstadt in czechoslovakia? what happens? how does vladek find out?. Which of the following sets of numbers is a Pythagorean triple?42, 40, 8242, 40, 5841, 41, 5840, 41, 57HELP QUICK PLEEZ Pearl and Jason are working on homework together when Pearl says, "got x = 6 and x=13 as the solutions, but it looks like you got something different. Which solutions are right?"I found a mistake," says Jason. Where did Pearl make a mistake? Explain why it is incorrect. Pearl's work is shown below.